0% found this document useful (0 votes)
680 views15 pages

Analysis 2 I

This document contains a student's assignment submission for a real analysis course. The assignment contains solutions to 13 problems from the textbook. The problems cover topics such as measurable functions, Lebesgue integration, and convergence theorems. The student provides detailed solutions and explanations for each problem.

Uploaded by

Miliyon Tilahun
Copyright
© © All Rights Reserved
We take content rights seriously. If you suspect this is your content, claim it here.
Available Formats
Download as PDF, TXT or read online on Scribd
0% found this document useful (0 votes)
680 views15 pages

Analysis 2 I

This document contains a student's assignment submission for a real analysis course. The assignment contains solutions to 13 problems from the textbook. The problems cover topics such as measurable functions, Lebesgue integration, and convergence theorems. The student provides detailed solutions and explanations for each problem.

Uploaded by

Miliyon Tilahun
Copyright
© © All Rights Reserved
We take content rights seriously. If you suspect this is your content, claim it here.
Available Formats
Download as PDF, TXT or read online on Scribd
You are on page 1/ 15

Assignment I

REAL ANALYSIS II


April 13, 2016

Submitted to

Dr. Legesse Lemecha

Name
ID No
1.
Miliyon Tilahun . . . . . . . . . . . . . . . . . . . . . . . . . . . . . . . . . . . . . . . . . GSR/1401/08

Problems from Roydens Real Analysis Chapter 4


1. R(#9) Let E have measure zero. Show that if f is a bounded function on E, then f is measurable and
f = 0.
E
Solution. Let F be a measurable subset of R. Then f 1 (F ) is a subset of E, and since subsets of
measure zero sets have measure zero, f 1 (F ) has measure zero. In particular f 1 (F ) is measurable, so
f is measurable. Since f is bounded on E, for some constants and we have f . Moreover,
by monotonicity of Lebesgue integral we have
Z
m(E)
f m(E)
E

But m(E) = 0, implying


Z
f = 0.
E

2. (#10) Let f be a bounded


R
Rmeasurable function on a set of finite measure E. For a measurable subset
A of E, show that A f = E f A .
Solution. For a bounded measurable function f on a set of finite measure E we have
Z
Z
Z
f=
f+
f.
AB

where A and B are disjoint measurable subsets of E.


Since A E, E = A (E \ A). Clearly, A and E \ A are disjoint. Since A is a measurable subset of
E we have E \ A is measurable because the set E = A (E \ A) is measurable. Therefore
Z
Z
Z
Z
Z
f A =
f A +
f A =
f A =
f
E

E\A

This is because f A = f on A, and f A = 0 on E \ A.


3. (#13) Does the Bounded Convergence Theorem hold if m(E) < but we drop the assumption that
the sequence (|fn |) is uniformly bounded on E?
Solution. No it does not hold! Take fn = n2 [0, n1 ] which is not uniformly bounded. Then we see
that fn f 0. But


Z
Z
1
1
lim
fn = lim
n2 [0, n1 ] = lim n2 m 0,
= lim n2 =
n
n
n
n
n
n
This concludes

Z
fn = =
6 0=

lim

4. (#17) Let E be a set of measure zero and define f on E. Show that

R
E

f = 0.

Solution. By definition we have


Z

Z
f = sup
hf

h
E

where f is a nonnegative measurable function on measurable set E and h is a bounded measurable


R
function such that m{x : h(x)
R 6= 0} is finite. If h is such a function and m(E) = 0, we have E h = 0
by problem 1(#9). Hence E f = 0.
2

5. (#21) Let the function f be nonnegative and integrable over


R E and > 0. Show there is a simple
function on E that has finite support, 0 f on E and E |f | < R . If E is a closed, bounded
interval, show there is a step function h on E that has finite support and E |f h| < .
Solution. Let > 0. We know by the definition of the integral for non-negative measurable functions
that there exists a bounded, measurable function of finite support h f , such that
Z
Z
h>
f /2.
E

We also know that for any given bounded, measurable function h with finite support, there exists a
simple function h with the same support as h such that
Z
Z
>
h /2.
E

Therefore we see that there is a simple function with finite support such that f , and
Z
(f ) <
E

In the case that E is a closed and bounded interval, let =


that
Z
(f ) < /2.

Pn

k=1 ck Ek

be a simple function such

E
k
For each k, since m(Ek ) < , there exists a finite collection of disjoint open intervals {Iik }ni=1
such
that



Z
nk
nk
X
[


k
E

=
m
E

I
k
k
Ii
i < /2n.
k

i=1

i=1

Moreover, since E is a closed bounded interval, we may assume that each Iik is contained inside E, but
may no longer be open. Define a function by
=

nk
n X
X

ck Iik .

k=1 i=1

Note that this function is a step function even though some of the intervals may overlap, since any
finite collection of intervals that cover E define a partition of E by ordering their endpoints. It follows
that
Z
Z
nk
n
X
X


| |
ck Ek
Iik /2.
E

k=1

i=1

Therefore is a step function such that,


Z
| f | <
E

6. (#22) Let {fn } be a sequence of nonnegative measurable functions on R that converges pointwise on
R to f and f be integrable over R. Show that
Z
Z
Z
Z
if
f = lim
fn , then
f = lim
fn for any measurable set E.
R

R
R
R
Solution. Let E be a measurable set. Using the fact that R f = E f + R\E f , Fatous lemma on
R \ E implies
Z
Z
Z
Z
f lim inf
fn =
f lim sup
fn .
R\E

Therefore

R\E

Z
f lim sup
n

fn

(1)

fn .

(2)

However Fatous Lemma on E gives


Z

Z
f lim inf
n

By using the inequalities (1) and (2) we conclude that


Z
Z
f = lim
fn
n

7. (#25) Let {fn } be a sequence of nonnegative measurable functions on E that converges pointwise on
E to f . Suppose fn f on E for each n. Show that
Z
Z
fn =
f.
lim
n

R
Solution. Since fn f on E for each n,we have E fn E f , therefore
Z
Z
lim sup
fn
f.
n

However, since {fn } are nonnegative, Fatous Lemma implies


Z
Z
f lim inf
fn .
n

(3)

(4)

Thus, using (3) and (4) we arrive at the conclusion


Z
Z
fn =
f
lim
n

8. (#26) Show that the Monotone Convergence Theorem may not hold for decreasing sequences of functions.
Solution. Let f : R R be the zero function, and consider the sequence {fn } denoted by
fn (x) = [n,) (x)
Note that fn is a simple function and [n, ) is measurable. By definition
Z
fn = m([n, )) =
for every n Z+ . Now we will show that fn is monotone decreasing. Let n Z+ and x R. If
x < n + 1, then x
/ [n + 1, ) so
fn+1 (x) = 0 fn (x).
4

If x n + 1, then x [n + 1, ) and x [n, ), so


fn+1 (x) = 1 1 = fn (x).
This establishes that fn+1 fn for all n Z+ . In other words {fn } is a monotone decreasing sequence
of functions. Hence
Z
Z
Z
fn
f = 0 = 0 6= = lim = lim
n

showing that the Monotone Convergence Theorem need not hold for decreasing sequences.
R n+1
9. (#29) For a measurable function f on [1, ) which is bounded on bounded sets, define an =
f
n
P
for each natural number n. Is it true that f is integrable over [1, ) if and only if P
the series n=1 an

converges? Is it true that f is integrable over [1, ) if and only if the series
n=1 an converges
absolutely?
Solution. NO!
Take

(1)n
, n x n + 1, n 1
n
This function is not Lebesgue integrable though the series is convergent.
For the second one take,
(
1,
n x n + 21 for some n 1.
f (x) =
1, otherwise
f (x) =

Then an = 0 for every n, hence

n=1

|an | converges. But f is not Lebesgue integrable.

10. (#34) Let f be a nonnegative measurable function on R. Show that


Z n
Z
lim
f=
f.
n

Solution. Let fn = f [n,n] , then fn is nonnegative, monotone and fn f pointwise. By monotone


convergence theorem we have
Z n
Z
Z
lim
f = lim
fn =
f.
n

11. (#35) Let f be a real-valued function of two variables (x, y) that is defined on the square Q =
{(x, y)|0 x 1, 0 y 1} and is a measurable function of x for each fixed value of y. Suppose for
each fixed value of x, limy0 f (x, y) = f (x) and that for all y, we have |f (x, y)| g(x), where g is
integrable over [0, 1]. Show that
1

Z
lim

y0

Z
f (x, y)dx =

f (x)dx.
0

Also show that if the function f (x, y) is continuous in y for each x, then
Z
h(y) =

f (x, y)dx
0

is a continuous function of y.

Solution. It suffices to consider a sequence {yn } [0, 1] such that yn 0. Define fn (x) = f (x, yn ).
Then since fn f pointwise and |fn | g, Lebesgue dominated convergence gives
Z 1
Z 1
Z 1
f (x)dx.
fn (x)dx =
f (x, yn )dx = lim
lim
n

Since this is true for every such sequence {yn }, we have


Z 1
Z
f (x, y)dx =
lim
y0

f (x)dx.

Now fix y [0, 1] and suppose that {yn } [0, 1] such that yn y. Define fn (x) = f (x, yn ). Since
f (x, y) is continuous, fn (x) f (x, y) for all x [0, 1]. Once again, since |fn (x)| g(x) dominated
convergence gives
Z 1
Z 1
fn (x)dx =
f (x, y)dx = h(y).
lim h(yn ) = lim
n

Therefore h is continuous.
12. (#36) Let f be a real-valued function of two variables (x, y) that is defined on the square Q =
{(x, y)|0 x 1, 0 y 1} and is a measurable function on x for each fixed value of y. For each
(x, y) Q let the partial derivative f /y exist. Suppose there is a function g that is integrable over
[0, 1] and such that


f

(x, y) g(y) for all (x, y) Q.
y

Prove that
d
dy

Z

 Z
f (x, y)dx =

f
(x, y)dx for all y [0, 1].
y

Solution. Fix y0 [0, 1) and define for n 1,


Dn f (x) = n(f (x, y0 + 1/n) f (x, y0 )).
f
y (x, y0 ) pointwise. Also
|Dn f (x)| = | f
y (x, )| g(x). We

Since the derivative exists, Dn f (x)

by the Mean Value Theorem, there is

a [y0 , y0 + 1/n] such that


conclude by the Lebesgue Dominated
Convergence Theorem that
Z 1

Z 1
Z 1
Z 1
f
d
(x, y0 )dx = lim
Dn f (x)dx = lim Dn
f (x, y0 )dx =
f (x, y0 )dx .
n 0
n
dy 0
0 y
0
Since this is true for all y0 [0, 1) we have the result.
13. (#44) Let f be integrable over R and > 0. Establish the following three approximation properties.
R
(i) There is a simple function on R which has finite support and R |f | < .
R
(ii) There is a step function s on R which vanishes outside a closed, bounded interval and R |f s| < .
(Hint: Apply part (i) and Problem 18 of Chapter 3.)
R
(iii) There is a continuous function g on R which vanishes outside a bounded set and R |f g| < .
Solution. (i) If f is non-negative we may find an increasing sequence of non-negative simple functions
{n } with finite support such that n f pointwise. It follows by monotone convergence that we can
find a such that
Z
Z
|f | =
R

f < .
R

For
f we write Rf = f + f , and find 1 and 2 simple and of finite support such that
R general
+
|f 1 | < /2 and R |f 2 | < /2. Since f + and f have disjoint support, we see that 1
R
6

and 2 must also have disjoint support, therefore = 1 2 is also simple with finite support and it
follows that
Z
Z
Z
+
|f |
|f 1 | + |f 2 | < .
R

(ii) By part (i), since we can approximate by simple functions, by the triangle inequality it suffices to
show that the characteristic function E of a bounded measurable set E can be approximated by step
functions. Note S
that since E is measurable, we can find a disjoint collection of open intervals {Ik }
k=1

such that O = k=1 Ik , and


Sm(O \ E) < /2. Since O must have finite measure we can find an N
large enough such that m( k=N +1 Ik ) < /2. Therefore
s=

N
X

In

k=1

is a step function and


Z
|E s|
R

N Z
X
k=1

|EIk Ik | +

k=N +1

EIk



 [
[

N
Ik E
Ik \ E +m
m
k=N +1

k=1

 [


m(O \ E) + m
Ik < .
k=N +1

(iii) Using part (ii), once again we see by the triangle inequality that it suffices to show that any
characteristic function of a bounded interval [a,b] can be approximated by a continuous function. Let
g be the continuous function which is 1 on [a + /2, b /2] and linearly interpolated to 0 outside of
[a, b], then
Z
|[a,b] g| < m([a, a + /2) (b /2, b]) = .
R

14. (#46) Let f be integrable over (, ). Show that


Z
lim
f (x) cos nxdx = 0.
n

we know there exists a step function s, vanishing


Solution. Using the result from problem 13(#44),
R
outside of a closed bounded interval such that R |f s| < /2. Let s be the step function which we
write in canonical form as
K
X
s=
sk (ak ,bk ) ,
k=1

where {(ak , bk )} are a disjoint collection of bounded open intervals and {sk } are distinct. Note that it
doesnt matter that we dont define s at the end points of the intervals since they are a set of measure
0. We see that
Z
X
Z bk

K








s(x)
cos
nxdx
|s
|
cos
nxdx
k


ak

k=1

K
X
|sk |
k=1

| sin nbk sin nak |

2K max{|si |}
n

Therefore if n > N 4K max{si }/, we conclude


Z
Z
Z





|f (x) s(x)|dx +
f (x) cos nxdx



s(x) cos nxdx

< /2 + /2 = .

15. (#47) Let f be integrable over (, ).


(i) Show that for each t,

f (x)dx =

f (x + t)dx.

(ii) Let g be a bounded measurable function on R. Show that


Z
g(x) [f (x) f (x + t)]dx = 0.
lim
t0

(Hint: First show this, using uniform continuity of f on R, if f is continuous and vanishes outside a
bounded set. Then use the approximation property (iii) of Problem #44.)
P
Solution. (i) Note that this result is true for simple functions. Let = k=1 ci Ek be simple, then
since E (x + t) = Et (x), we haven by the translation invariance of Lebesgue
Z
Z
n
n
X
X
ck m(Ek ) =
(x)dx.
ck m(Ek t) =
(x + t)dx =
R

k=1

k=1

For the case of integrable f , we may restrict ourselves to f non-negative, since we may always write
any integrable f as f = f + f , where f + (x) = max{f (x), 0} and f (x) = max{f (x), 0}. For such
a non-negative f , we know that there is a sequence of simple functions with finite support {n }
n=1
such that n f with n converging to f pointwise and monotonically. In particular (+t) f (+t)
monotonically for every fixed t. It follows from the monotone convergence theorem that
Z
Z
Z
Z
n (x)dx =
f (x)dx.
n (x + t)dx = lim
f (x + t)dx = lim
n

(ii) Let M < be a constant such that |g| M . Since f is integrable, we know byRProblem 13 (#44)
that for every > 0 there exists h, continuous and of bounded support such that R |f h| < /2M .
It follows that h uniformly continuous and bounded by some constant N . Let {tn } be any sequence
tn 0 and define hn (x) = |h(x) h(x + tn )|. Then hn 0 pointwise and 0 hn (x) 2N . It follows
by the bounded convergence theorem that
Z
lim
|f (x) f (x + tn )| = 0.
n

Therefore
Z

Z
Z
Z


g(x)[f (x)f (x+tn )]dx M
|f
(x)h(x)|dx+M
|f
(x+t
)h(x+t
)|dx+M
|f (x)f (x+tn )|dx
n
n


R

By part i) and the way that h was chosen, the sum of the first two terms on the righthand side above
are bounded by . Taking the lim supn of both sides, we obtain
Z

Z



lim sup g(x) [f (x) f (x + t)]dx < + M lim
|f (x) f (x + tn )|dx = .
n

Since this is true for every > 0 and every {tn }, tn 0, we conclude
Z
lim g(x) [f (x) f (x + t)]dx.
t0

16. (#49) Let f be integrable over R. Show that the following four assertions are equivalent:
(i) f = 0 a.e on R.
R
(ii) R f g = 0 for every bounded measurable function g on R.
R
(iii) A f = 0 for every measurable set A.
R
(iv) O f = 0 for every open set O.
Solution. (1) (2)
If f = 0 a.e. on R, then m(E0 ) = 0, where
E0 = {x R : f (x) 6= 0}.
R
Since f is integrable over R, we have R |f |d < and g : R R being bounded and measurable so
|g(x)| < M for all x, then the function |f g| is bounded from above by M |f |, so we have
Z
Z
Z
|f g|d
M |f | = M
|f | < .
R

Thus f g is integrable and


Z

fg =
R

fg +

f g,
E0

RE0

But since f = 0 a.e on R and m(E0 ) = 0, we have


Z
Z
f g = 0, and

f g = 0,

E0

RE0

Thus,
Z
f g = 0,
R

as required.
(2) (3)
For a measurable set A, let g = A . Clearly, g is measurable and bounded. Therefore,
Z
Z
Z
0=
fg =
f A =
f = 0,
R

as required.
(3) (4)
Since every open set is measurable, then if (3) holds, we have
Z
f = 0 for every open set O,
O

as required.
(4) (1)
Let {Un }nN be a countable collection of open sets. Then,
\
G=
On ,
nN

T
is a G set with On = kn Uk , where each Uk is an open set. Thus, {On }nN is a countable descending
collection of open sets, i.e.
i N : Oi Oi+1 .
9

Now, for On being open, we have

Z
f = 0,
On

and by the continuity of integration,


Z
Z
f= T
G

Z
f = lim

nN

On

f = 0.
On

Since every measurable set E R is of the form G E0 , where G is a G subset of R and m(E0 ) = 0,
so that
Z
f = 0 for each measurable set E R.
E

Now, define
E + = {x R : f (x) 0} and E = {x R : f (x) 0},
then E + and E are measurable subsets of R and thus,
Z
Z
+
f =
f = 0,
E+

and

(f ) =

Z
f = 0,
E

where f + = max{f, 0} and f = max{f, 0}. Finally we use the fact (proposition 9 chapter 4) that
Z
f = 0 if and only if f = 0 a.e
E

so that f + and f vanishes almost everywhere in R and so does f as required.

Problems from Roydens Real Analysis Chapter 6


17. (#16) Let g be integrable over [a, b]. Define the antiderivative of g to be the function f defined on
[a, b] by
Z x
f (x) =
g
for all x [a, b].
a

Show that f is differentiable almost everywhere on (a, b).


Solution. Let us write g as a positive and negative part that is
g = g+ g
where
g + = max{f (x), 0}
g = max{f (x), 0}
Both g + and g are non negative. Then
Z

f (x) =
a

g+

Since f is the difference of two monotone increasing functions, then it is of bounded variation and f is
differentiable a.e on (a, b) by the corollary of Jordans theorem.

10

18. (#24) Show that for f defined in the last remark of this section, f 0 is not integrable over [0, 1].
Solution. The function in the remark is
(
f (x) =

x2 sin(1/x2 )
0

for 0 < x 1,
for x = 0.

To show f 0 is not integrable on [0, 1] it suffices to show that f is not of bounded variation on [0, 1].
Since every absolutely continuous function is of bounded variation it follows that f is not absolutely
continuous on [0, 1]. Hence by the Fundamental Theorem of Calculus for Lebesgue integrals we conclude
that f 0 is not integrable.
We show that f is not of bounded variation by showing that the total variation of f is bounded below
by a divergent series.
To make a suitable partition of [0, 1], note first that sin(x) = 1 for all x = 2 + 2k with k N, and
(1+2k)
with k N. Hence
sin(x) = 1 for all x = 3
2 + 2k with k N. So | sin(x)| = 1 for all x =
2
q
1
2
| sin( x2 )| = 1 for all x = (1+2k) for k N, and the value is negative if k is odd and positive if k is
even. So for each N N we can take a partition {xk }N
k=0 of [0, 1] so that
s
2
for all k = 0, . . . , N.
xk =
(1 + 2k)
Now f (xk ) and f (xk1 ) always have the opposite sign, so it follows that
T01 (f )

N
X

|f (xk ) f (xk1 )|

k=1

N 
X
k=1

N
X
k=1

N
X
k=1

2
2

(2k + 1) (2k 1)

2
(2k + 1)
N
2
2 X1
=
.
(3k)
3
k
k=1

This inequality holds for all N N, so we get that


T01 (f )

2 X1
= .
3
k
k=1

So f is not of bounded variation on [0, 1].


19. (#29) (a) Define
(
x2 cos(1/x2 )
f (x) =
0

if x 6= 0, x [1, 1]
if x = 0.

Is f of bounded variation on [1, 1]?


(b) Define
(
g(x) =

x2 cos(1/x)
0

Is g of bounded variation on [1, 1]?

11

if x 6= 0, x [1, 1]
if x = 0.

Solution. (a) We show that f is not of bounded variation by showing that the total variation of f is
bounded below by a divergent series.
Since f (x) = f (x) for all x [1, 1] it suffices to show that the total variation of f in [0, 1] is not
finite. To make a suitable partition of [0, 1], note first that cos(x) = 1 for all x = 2k with k N,
and cos(x) = 1 for all xq= (2k + 1) with k N. So | cos(x)| = 1 for all x = k with k N. Hence
| sin( x12 )| = 1 for all x =

1
k

for k N, and the value is negative if k is odd and positive if k is even.

So for each N N we can take a partition {xk }N


k=0 of [0, 1] so that
r
1
xk =
for all k = 0, . . . , N.
k
Now f (xk ) and f (xk1 ) always have the opposite sign, so it follows that
T01 (f )

N
X

|f (xk ) f (xk1 )|

k=1


N 
X
1
1

=
k (k 1)
k=1

N
N
X
1X1
1
=
.
k

k
k=1

k=1

This inequality holds for all N N, so we get that


T01 (f )

1X1
= .

k
k=1

So f is not of bounded variation on [1, 1].


(b) Note that for x 6= 0, we have
g 0 (x) = 2x cos(1/x) sin(1/x)
and x = 0,
h2 cos( h1 )
cos h
g(h) g(0)
= lim
= lim h cos(1/h) = lim
=0
h0
h0
h h
h0
h
h

g 0 (0) = lim

Thus, the derivative of g exists for all x [1, 1] and it is bounded by


|g 0 (x)| = |2x cos(1/x) sin(1/x)|
= 2|x|| cos(1/x)| + | sin(1/x)|
2+1=3
Since g(x) 0 as x 0 then g is continuous so the standard trick to find the Lipschitz constant of
g is to use the Mean Value Theorem(MVT). For any x < y in [1, 1], by MVT there exists a point
(x, y) such that
|g(x) g(y)| = |g 0 ()||x y|
Now since the derivative of g is bounded from above by 3, this implies
g(x) g(y) 3|x y|,

for all x, y [1, 1]

Therefore, g is Lipschitz. Since every Lipschitz function is of bounded variation g is of bounded


variation.

12

20. (#33) Let {fn } be a sequence of real-valued functions on [a, b] that converges pointwise on [a, b] to the
real-valued function f . Show that
T V (f ) lim inf T V (fn ).
Solution. Let P be any partition of [a, b], then since V (fn , P ) only depends on fn at a finite number
of points,
V (f, P ) = lim V (fn , P ).
n

Furthermore V (fn , P ) T V (fn ) and therefore


V (f, P ) = lim V (fn , P ) lim inf T V (fn ).
n

Taking the sup over all partitions P gives


T V (f ) lim inf T V (fn ).
n

21. (#35) For and positive numbers, define the function f on [0, 1] by
(
x sin(1/x ) for 0 < x 1
f (x) =
0
for x = 0.
Show that if > , then f is of bounded variation on [0, 1], by showing that f 0 is integrable over
[0, 1]. Then show that if , then f is not of bounded variation on [0, 1].
Solution. If > , then
f 0 (x) = x1 sin(1/x ) x1 cos(1/x ).
Since f is C 1 and bounded on (0, 1) we can use the fundamental theorem of calculus for Riemann
integrals to conclude that for any partition P
V (f, P ) =

n
X

Z
|f (xk ) f (xk1 )|

Z
T V (f )

|f 0 |

|f 0 |

k=0

and therefore

x1 + x1 < ,

since > 0 and > 0.


If , choose a partition Pn = {0, an , an1 , . . . , a1 }, where

an =
then we see that
V (f, Pn ) =

n
2

1/

/
n 
X
k
k=1

Note that this series diverges as n since / 1. Therefore


lim V (f, Pn ) T V (f ) = .

13

22. (#49) Let f be continuous on [a, b] and differentiable almost everywhere on (a, b). Show that
Z b
f 0 = f (b) f (a)
a

if and only if

Z
Z b
lim Diff1/n f = lim
a


Diff1/n f

Solution. Since f is continuous and differentiable a.e. on (a, b), we have that



Z b
Diff1/n f = lim Av1/n f (b) Av1/n f (a) = f (b) f (a),
lim
n

and

Z b
a

 Z
lim Diff1/n f =

f0

for almost all x. The result then follows immediately.


23. (#52) Let f and g be absolutely continuous on [a, b]. Show that
Z b
Z
f g 0 = f (b)g(b) f (a)g(a)
a

f 0 g.

Solution. Since g is absolutely continuous {Diff1/n g} is uniformly integrable and therefore {(Diff1/n g)
f } is uniformly integrable. Thus
Z b
Z b
f g 0 = lim
f Diff1/n g.
n

however by direct computation we see that


Z b
Z b
Z
f Diff1/n g = n
f (x)g(x + 1/n)dx
a

b1/n


f (x + 1/n)g(x + 1/n)dx

a1/n

(f (x) f (x + 1/n))g(x + 1/n)dx + n

=n

f (x + 1/n)g(x + 1/n)dx

b1/n

Z
n

f (x + 1/n)g(x + 1/n)dx
a1/n

Z
= Av1/n (f g)(b) Av1/n (f g)(a)

Diff1/n f g.

(5)

Since f and g are continuous,


Av1/n (f g)(b) f (b)g(b), Av(f g)(a) f (a)g(a),

(6)

and f is absolutely continuous so {Diff1/n f g} is uniformly integrable, therefore


Z

Z
Diff1/n f g

f 0 g.

(7)

Hence using (5),(6) and (7) we conclude that


Z b
Z
f g 0 = f (b)g(b) f (a)g(a)
a

14

f 0 g.

24. (#53) Let the function f be absolutely continuous on [a, b]. Show that f is Lipschitz on [a, b] if and
only if there is a c > 0 for which |f 0 | c a.e. on [a, b].
Solution. Suppose f is Lipschitz, then there exists a constant c > 0 such that
|f (y) f (x)| c|y x|,

x, y [a, b].

|f (y) f (x)|
c
|y x|

Now
f 0 (x) = lim

yx

f (y) f (x)
yx




f (y) f (x)
|f (y) f (x)|
lim
|f 0 (x)| = lim
lim c = c
yx
yx
y x yx
|y x|
Thus, |f 0 | c on [a, b].
Conversely, suppose |f 0 | c on [a, b]. Since f is absolutely continuous on [a, b], then it is differentiable
a.e on (a, b). By MVT there exists (a, b) such that
|f (x) f (y)| |f 0 ()||x y| c|x y|
Hence, f is Lipschitz.

15

You might also like